Give a formula the open unit disk

Click For Summary
In the open unit disk D, the formula for the sum \(\sum_{n=0}^{\infty} (xy)^{n}\) is \(\frac{1}{1 - xy}\) when \(xy < 1\), which holds for all points in D. The confusion arises from the application of the geometric series formula, which states that the sum converges to \(\frac{1}{1-r}\) for \(|r|<1\). For part (b), the integral \(\int_{x}^{y} e^{-t} dt\) should be evaluated first, followed by applying the product rule to find the partial derivative with respect to \(y\). The discussion emphasizes understanding these mathematical concepts rather than memorizing solutions. Overall, clarity in the application of series and differentiation techniques is crucial for solving these types of problems.
nhartung
Messages
56
Reaction score
0

Homework Statement



In the open unit disk D = {(x,y)|x2 + y2 < 1} give a formula for:

(a) \frac{\partial \sum ^{inf}_{n=0} (xy) ^{n}}{\partial x}

(b) \frac{\partial (e^{x+y} \int ^{y}_{x} e^{-t} dt)}{\partial y}

Homework Equations





The Attempt at a Solution



Ok this is on my exam review sheet and he gave us the solutions to go along with it.. I don't know if I wasn't paying attention in class or what but I don't remember doing anything like this.

His first step for a is \sum ^{inf}_{n=0} (xy)^{n} = \frac{1}{1 - xy} if xy < 1 (But for all (x,y)\in D xy < 1

I'm already confused at this point. Can someone please let me know what is going on here? Thanks
 
Physics news on Phys.org
The sum of r^n is 1/(1-r) if |r|<1. Now put r=(xy). It's just a geometric series. For the second one just evaluate the integral and take the partial derivative using the product rule etc. There's nothing really special going on there.
 
Question: A clock's minute hand has length 4 and its hour hand has length 3. What is the distance between the tips at the moment when it is increasing most rapidly?(Putnam Exam Question) Answer: Making assumption that both the hands moves at constant angular velocities, the answer is ## \sqrt{7} .## But don't you think this assumption is somewhat doubtful and wrong?

Similar threads

Replies
3
Views
2K
Replies
5
Views
2K
  • · Replies 2 ·
Replies
2
Views
2K
  • · Replies 4 ·
Replies
4
Views
2K
  • · Replies 1 ·
Replies
1
Views
2K
Replies
2
Views
1K
  • · Replies 18 ·
Replies
18
Views
3K
  • · Replies 4 ·
Replies
4
Views
1K
  • · Replies 8 ·
Replies
8
Views
2K
  • · Replies 6 ·
Replies
6
Views
2K